Những câu hỏi liên quan
Huy bủh
Xem chi tiết
Akai Haruma
4 tháng 9 2021 lúc 23:08

Lời giải:

$\sin a+\cos a=1$

$\sin ^2a+\cos ^2a=1$

$\Rightarrow 2\sin a\cos a=(\sin a+\cos a)^2-(\sin ^2a+\cos ^2a)=1^2-1=0$

$\Rightarrow \sin a\cos a=0$

$\Rightarrow \sin a=0$ hoặc $\cos a=0$

Nếu $\sin a=0$ hoặc $\cos a=0$

Mà vì $a$ là góc nhọn nên $\sin a, \cos a< 1$ nên không tìm được góc $a$ thỏa mãn. 

Bình luận (0)
yêu húa
Xem chi tiết
Phùng Minh Quân
30 tháng 11 2019 lúc 18:26

\(\sin^4\alpha+\cos^4\alpha=\left(\sin^2\alpha+\cos^2\alpha\right)^2-2\sin^2\alpha.\cos^2\alpha=1-2.\frac{1}{4^2}=\frac{7}{8}\)

Bình luận (0)
 Khách vãng lai đã xóa
Phan Lê Kim Chi
Xem chi tiết
alibaba nguyễn
27 tháng 8 2021 lúc 16:34

a/ \(A=\frac{cot^2a-cos^2a}{cot^2a}-\frac{sina.cosa}{cota}\)

\(=\frac{\frac{cos^2a}{sin^2a}-cos^2a}{\frac{cos^2a}{sin^2a}}-\frac{sina.cosa}{\frac{cosa}{sina}}\)

\(=\left(1-sin^2a\right)-sin^2a=1\)

Bình luận (0)
 Khách vãng lai đã xóa
alibaba nguyễn
27 tháng 8 2021 lúc 16:38

b/ \(B=\left(cosa-sina\right)^2+\left(cosa+sina\right)^2+cos^4a-sin^4a-2cos^2a\)

\(=cos^2a-2cosa.sina+sin^2a+cos^2a+2cosa.sina+sin^2a+\left(cos^2a+sin^2a\right)\left(cos^2a-sin^2a\right)-2cos^2a\)

\(=2+\left(cos^2a-sin^2a\right)-2cos^2a\)

\(=2-sin^2a-cos^2a=2-1=1\)

Bình luận (0)
 Khách vãng lai đã xóa
alibaba nguyễn
27 tháng 8 2021 lúc 16:41

c/ \(C=sin^6x+cos^6x+3sin^2x.cos^2x\)

\(=\left(sin^2x+cos^2x\right)\left(sin^4x-sin^2x.cos^2x+cos^4x\right)+3sin^2x.cos^2x\)

\(=sin^4x-sin^2x.cos^2x+cos^4x+3sin^2x.cos^2x\)

\(=sin^4x+cos^4x+2sin^2x.cos^2x\)

\(=\left(sin^2x+cos^2x\right)^2=1\)

Bình luận (0)
 Khách vãng lai đã xóa
Quoc Tran Anh Le
Xem chi tiết
Hà Quang Minh
21 tháng 9 2023 lúc 22:42

a) Vì \(0<\alpha <\frac{\pi }{2} \) nên \(\sin \alpha  > 0\). Mặt khác, từ \({\sin ^2}\alpha  + {\cos ^2}\alpha  = 1\) suy ra

\(\sin \alpha  = \sqrt {1 - {{\cos }^2}a}  = \sqrt {1 - \frac{1}{{25}}}  = \frac{{2\sqrt 6 }}{5}\)

Do đó, \(\tan \alpha  = \frac{{\sin \alpha }}{{\cos \alpha }} = \frac{{\frac{{2\sqrt 6 }}{5}}}{{\frac{1}{5}}} = 2\sqrt 6 \) và \(\cot \alpha  = \frac{{\cos \alpha }}{{\sin \alpha }} = \frac{{\frac{1}{5}}}{{\frac{{2\sqrt 6 }}{5}}} = \frac{{\sqrt 6 }}{{12}}\)

b) Vì \(\frac{\pi }{2} < \alpha  < \pi\) nên \(\cos \alpha  < 0\). Mặt khác, từ \({\sin ^2}\alpha  + {\cos ^2}\alpha  = 1\) suy ra

       \(\cos \alpha  = \sqrt {1 - {{\sin }^2}a}  = \sqrt {1 - \frac{4}{9}}  = -\frac{{\sqrt 5 }}{3}\)

Do đó, \(\tan \alpha  = \frac{{\sin \alpha }}{{\cos \alpha }} = \frac{{\frac{2}{3}}}{{-\frac{{\sqrt 5 }}{3}}} = -\frac{{2\sqrt 5 }}{5}\) và \(\cot \alpha  = \frac{{\cos \alpha }}{{\sin \alpha }} = \frac{{-\frac{{\sqrt 5 }}{3}}}{{\frac{2}{3}}} = -\frac{{\sqrt 5 }}{2}\)

Bình luận (0)
Hà Quang Minh
21 tháng 9 2023 lúc 22:43

c) Ta có: \(\cot \alpha  = \frac{1}{{\tan \alpha }} = \frac{1}{{\sqrt 5 }}\)

Ta có: \({\tan ^2}\alpha  + 1 = \frac{1}{{{{\cos }^2}\alpha }} \Rightarrow {\cos ^2}\alpha  = \frac{1}{{{{\tan }^2}\alpha  + 1}} = \frac{1}{6} \Rightarrow \cos \alpha  =  \pm \frac{1}{{\sqrt 6 }}\)

Vì \(\pi  < \alpha  < \frac{{3\pi }}{2} \Rightarrow \sin \alpha  < 0\;\) và \(\,\,\cos \alpha  < 0 \Rightarrow \cos \alpha  = -\frac{1}{{\sqrt 6 }}\)

Ta có: \(\tan \alpha  = \frac{{\sin \alpha }}{{\cos \alpha }} \Rightarrow \sin \alpha  = \tan \alpha .\cos \alpha  = \sqrt 5 .(-\frac{1}{{\sqrt 6 }}) = -\sqrt {\frac{5}{6}} \)

d) Vì \(\cot \alpha  =  - \frac{1}{{\sqrt 2 }}\;\,\) nên \(\,\,\tan \alpha  = \frac{1}{{\cot \alpha }} =  - \sqrt 2 \)

Ta có: \({\cot ^2}\alpha  + 1 = \frac{1}{{{{\sin }^2}\alpha }} \Rightarrow {\sin ^2}\alpha  = \frac{1}{{{{\cot }^2}\alpha  + 1}} = \frac{2}{3} \Rightarrow \sin \alpha  =  \pm \sqrt {\frac{2}{3}} \)

Vì \(\frac{{3\pi }}{2} < \alpha  < 2\pi  \Rightarrow \sin \alpha  < 0 \Rightarrow \sin \alpha  =  - \sqrt {\frac{2}{3}} \)

Ta có: \(\cot \alpha  = \frac{{\cos \alpha }}{{\sin \alpha }} \Rightarrow \cos \alpha  = \cot \alpha .\sin \alpha  = \left( { - \frac{1}{{\sqrt 2 }}} \right).\left( { - \sqrt {\frac{2}{3}} } \right) = \frac{{\sqrt 3 }}{3}\)

Bình luận (0)
ღ๖ۣۜVүү๖ۣۜLσηεlүүღ
Xem chi tiết
hanvu
13 tháng 2 2022 lúc 15:39

\(tan\alpha=\dfrac{1}{3}\Rightarrow\dfrac{sin\alpha}{cos\alpha}=\dfrac{1}{3}\Rightarrow cos\alpha=3sin\alpha\)

Thay cosa=3sina vào A, được:

\(A=\dfrac{sin^2a+9sin^2a}{sin^2a+9sin^2a+6sin^2a}=\dfrac{10sin^2a}{16sin^2a}=\dfrac{5}{8}\)

Bình luận (0)
Sách Giáo Khoa
Xem chi tiết
Thien Tu Borum
24 tháng 4 2017 lúc 13:56

Hướng dẫn giải:

a) tgα=ABAC=AB⋅BCAC⋅BCtgα=ABAC=AB⋅BCAC⋅BC

⇒tgα=ABBC÷ACBC=sinαcosα⇒tgα=ABBC÷ACBC=sinαcosα

tgα⋅cotgα=ABAC⋅ACAB=1tgα⋅cotgα=ABAC⋅ACAB=1

cotgα=1tgα=1sinαcosα=cosαsinαcotgα=1tgα=1sinαcosα=cosαsinα

b) sin2α+cos2α=AB2BC2+AC2BC2=BC2BC2=1sin2α+cos2α=AB2BC2+AC2BC2=BC2BC2=1

Nhận xét: Ba hệ thức tgα=sinαcosαtgα=sinαcosα

cotgα=cosαsinα;sin2α+cos2α=1cotgα=cosαsinα;sin2α+cos2α=1 là những hệ thức cơ bản bạn cần nhớ để giải một số bài tập khá

Bình luận (0)
Nguyễn Trần Thành Đạt
24 tháng 4 2017 lúc 13:56

a) tgα=ABAC=AB⋅BCAC⋅BCtgα=ABAC=AB⋅BCAC⋅BC

⇒tgα=ABBC÷ACBC=sinαcosα⇒tgα=ABBC÷ACBC=sinαcosα

tgα⋅cotgα=ABAC⋅ACAB=1tgα⋅cotgα=ABAC⋅ACAB=1

cotgα=1tgα=1sinαcosα=cosαsinαcotgα=1tgα=1sinαcosα=cosαsinα

b) sin2α+cos2α=AB2BC2+AC2BC2=BC2BC2=1sin2α+cos2α=AB2BC2+AC2BC2=BC2BC2=1

Nhận xét: Ba hệ thức tgα=sinαcosαtgα=sinαcosα

cotgα=cosαsinα;sin2α+cos2α=1cotgα=cosαsinα;sin2α+cos2α=1 là những hệ thức cơ bản bạn cần nhớ để giải một số bài tập khác.



Bình luận (0)
Nhật Linh
24 tháng 4 2017 lúc 13:56

2016-11-05_155403

Xét tam giác ABC vuông tại A, có góc B = α

a) 2016-11-05_1555272016-11-05_1555502016-11-05_155620

d) Tam giác ABC vuông tại A nên theo định lý pytago có:

2016-11-05_155801

Vậy: sin²a + cos²a = 1

Bình luận (0)
đỗ duy
Xem chi tiết
Nguyễn Huệ Lam
23 tháng 9 2018 lúc 15:54

Ta có:

\(sin^2a+cos^2a=1\Leftrightarrow sin^2a+\left(\frac{1}{3}\right)^2=1\Leftrightarrow sin^2a=\frac{8}{9}\Rightarrow sina=\frac{2\sqrt{2}}{3}.\)

\(B=\frac{sin\alpha-3cosa}{sina+2cosa}=\frac{\frac{2\sqrt{2}}{3}-3.\frac{1}{3}}{\frac{2\sqrt{2}}{3}+2.\frac{1}{3}}=\frac{7-5\sqrt{2}}{2}\)

Bình luận (0)
Quoc Tran Anh Le
Xem chi tiết
Hà Quang Minh
21 tháng 9 2023 lúc 22:32

+) Nửa đường tròn đơn vị: nửa đường tròn tâm O, bán kính R = 1 nằm phía trên trục hoành (H.3.2).

+) Với mỗi góc \(\alpha ({0^o} \le \alpha  \le {180^o})\)có duy nhất điểm \(M({x_0};{y_0})\) trên nửa đường tròn đơn vị nói trên để \(\widehat {xOM} = \alpha .\) Khi đó:

\(\sin \alpha  = {y_0}\) là tung độ của M

\(\cos \alpha  = {x_0}\) là hoành độ của M

\(\tan \alpha  = \frac{{\sin \alpha }}{{\cos \alpha }} = \frac{{{y_0}}}{{{x_0}}}(\alpha  \ne {90^o})\)

\(\cot \alpha  = \frac{{\cos \alpha }}{{\sin \alpha }} = \frac{{{x_0}}}{{{y_0}}}(\alpha  \ne {0^o},\alpha  \ne {180^o})\)

Bình luận (0)
Nguyễn Đình Hồng Minh
Xem chi tiết
Nguyễn Huệ Lam
Xem chi tiết
alibaba nguyễn
26 tháng 7 2017 lúc 9:40

Ta có:

\(\hept{\begin{cases}3sina+cosa=2\\sin^2a+cos^2a=1\end{cases}}\)

\(\Leftrightarrow\hept{\begin{cases}cosa=2-3sina\left(1\right)\\sin^2a+\left(2-3sina\right)^2=1\left(2\right)\end{cases}}\)

\(\left(2\right)\Leftrightarrow10sin^2a-12sina+3=0\)

\(\Leftrightarrow\orbr{\begin{cases}sina=\frac{3}{5}+\frac{\sqrt{6}}{10}\\sina=\frac{3}{5}-\frac{\sqrt{6}}{10}\end{cases}}\)

\(\Rightarrow\orbr{\begin{cases}cosa=\frac{1}{5}-\frac{3.\sqrt{6}}{10}\left(l\right)\\cosa=\frac{1}{5}+\frac{3.\sqrt{6}}{10}\end{cases}}\)

Thế vô tính tiếp

Bình luận (0)